Đến nội dung

nam8298 nội dung

Có 158 mục bởi nam8298 (Tìm giới hạn từ 05-05-2020)



Sắp theo                Sắp xếp  

#483089 Toán khó

Đã gửi bởi nam8298 on 14-02-2014 - 19:27 trong Số học

xét các trường hợp số dư của a và b khi chia cho 5 thì S chia hết cho 5 khi a và b chia 5 dư 4 hay (a-b) chia 5 dư 0




#489251 Tìm min M=$x^{3}+y^{3}+z^{3}= 12$.

Đã gửi bởi nam8298 on 28-03-2014 - 19:55 trong Bất đẳng thức và cực trị

chắcđề yêu cầu tìm min của M

xét $(x+1)(x-2)^{2}\geq 0$

nhân bung ra .

tương tự cho y và z rồi cộng vế




#463670 Tìm max$A=(3x-1)(2y-1)(z-1)$.

Đã gửi bởi nam8298 on 11-11-2013 - 20:15 trong Bất đẳng thức - Cực trị

đặt x= a+$\frac{1}{3}$ ; y =b+1$\frac{1}{2}$ ; z=c+1 suy ra a;b;c > 0

thay vào giả thiết thứ 2 ta đc $\frac{1}{a+1}+\frac{1}{b+1}+\frac{1}{c+1}\geq 2$ suy ra $\frac{a}{a+1}+\frac{b}{b+1}+\frac{c}{c+1}\leq 1$

ta có $\frac{1}{a+1}= 1-\frac{a}{a+1}\geq \frac{b}{b+1}+\frac{c}{c+1}\geq 2\sqrt{\frac{bc}{(b+1)(c+1)}}$

 chứng minh tương tự rồi nhân theo vế ta đc abc $\leq \frac{1}{8}$  từ đó tìm đc max A




#476384 $\Sigma \frac{a^{2}b}{4-bc}...

Đã gửi bởi nam8298 on 09-01-2014 - 20:31 trong Bất đẳng thức - Cực trị

Ta có bổ đề sau $a^{2}b+b^{2}c+c^{2}a+abc\leq 4$ suy ra $4-a^{2}b-b^{2}c-c^{2}a\geq abc$

BĐT tương đương $4-a^{2}b-b^{2}c-c^{2}a\geq \sum \frac{a^{2}b^{2}c}{4-bc}$

ta chứng minh abc $\geq \sum \frac{a^{2}b^{2}c}{4-bc}$

đặt a+b+c =p      ab+bc+ca =q          abc =r

tương đương $16-8q+q^{2}-r \geq 0$

mà $q^{2}\geq 9r$ nên ta chứng minh 16-8q+$q^{2}$ -$\frac{q^{2}}{9}$ $\geq 0$ tương đương (q-3)(q-6)$\geq 0$ (luôn đúng)




#458428 CMR $\sum \frac{a^{3}}{\sqrt...

Đã gửi bởi nam8298 on 18-10-2013 - 20:17 trong Bất đẳng thức và cực trị

Cho a,b,c >0 và $\sum a^{4}\geq \sum a^{3}$  CMR  $\sum \frac{a^{3}}{\sqrt{b^{4}+b^{2}c^{2}+c^{4}}}\geq \sqrt{3}$




#473774 Giải phương trình sau: 1.$16x^4+5=6\sqrt[3]{4x^3+x}$

Đã gửi bởi nam8298 on 29-12-2013 - 19:43 trong Phương trình - hệ phương trình - bất phương trình

2.áp dụng Am-Gm ta có $2\sqrt{10-x}=\frac{2}{3}\sqrt{9(10-x)}\leq \frac{(19-x)}{3} 

\sqrt[3]{4+4x}= \sqrt[3]{2.2.(x+1)}\leq \frac{5+x}{3}$

cộng vào ta đc VT<= VP

vậy pt có nghiệm x=1




#473772 Giải phương trình sau: 1.$16x^4+5=6\sqrt[3]{4x^3+x}$

Đã gửi bởi nam8298 on 29-12-2013 - 19:40 trong Phương trình - hệ phương trình - bất phương trình

1.theo đề bài thì x>o....áp dụng AM-GM ta có $6\sqrt[3]{4x^{3}+x}=3\sqrt[3]{2.4x.(4x^{2}+1)}\leq 4x+3+4x^{2}\leq 16x^{4}+5$

vậy x=0,5 là nghiệm




#477095 $\frac{x^2}{(ay+bz)(az+by)}+\frac{y^2...

Đã gửi bởi nam8298 on 13-01-2014 - 19:30 trong Bất đẳng thức và cực trị

ta chỉ cần chứng minh BĐT trong trường hợp a,b >0

áp dụng AM-GM ta có $4(ay+bz)(az+by)\leq (a+b)^{2}(y+z)^{2}$

BĐT cần chứng minh khi đó là $\sum \frac{x^{2}}{(y+z)^{2}}\geq \frac{3}{4}$ (luôn đúng do có BĐT $\sum \frac{x}{y+z}\geq \frac{3}{2}$




#476399 Tìm giá trị nhỏ nhất của biểu thức: $P=\frac{3a}{b+c...

Đã gửi bởi nam8298 on 09-01-2014 - 21:07 trong Bất đẳng thức và cực trị

P +12 =$\frac{3(a+b+c)}{b+c}+\frac{4(a+b+c)}{c+a}+\frac{5(a+b+c)}{a+b}=(a+b+c)(\frac{3}{b+c}+\frac{4}{c+a}+\frac{5}{a+b})=\frac{1}{2}(b+c+c+a+a+b)(\frac{3}{b+c}+\frac{4}{c+a}+\frac{5}{a+b})\geq \frac{(\sqrt{3}+\sqrt{4}+\sqrt{5})^{2}}{2}$

suy ra min P




#460926 $\frac{bc}{3a^2+b^2+c^2}+\frac{ca...

Đã gửi bởi nam8298 on 30-10-2013 - 20:17 trong Bất đẳng thức - Cực trị

theo mình đánh giá thế này $\frac{bc}{3a^{2}+b^{2}+c^{2}}\leq \frac{(b+c)^{2}}{12a^{2}+2(b+c)^{2}}$   

chuẩn hóa a+b+c =3 .sau đó dùng ước lượng là đc




#456207 $\frac{1}{a^{2}+b^{2}+2}+\frac{1}{b^{2}+c^{2}+2}+\frac{1}...

Đã gửi bởi nam8298 on 08-10-2013 - 21:20 trong Bất đẳng thức và cực trị

ta có $\sum \frac{1}{a^{2}+b^{2}+2}\leq \sum \frac{2}{(a+b)^{2}+4}= \sum \frac{2}{c^{2}-6c+13}$

ta chứng minh $\frac{2}{c^{2}-6c+13}\leq \frac{1}{4}+k(a-1)$ sau đó cộng theo vế đc đpcm




#456420 $\frac{1}{a^{2}+b^{2}+2}+\frac{1}{b^{2}+c^{2}+2}+\frac{1}...

Đã gửi bởi nam8298 on 09-10-2013 - 20:21 trong Bất đẳng thức và cực trị

chọn k = 1/8 sau đó biến đổi tương đương




#456888 $3(x^{2}+xy+y^{2})(y^{2}+yz+z^{2...

Đã gửi bởi nam8298 on 11-10-2013 - 20:28 trong Bất đẳng thức và cực trị

cho x,y,z là các số thực CMR:$3(x^{2}+xy+y^{2})(y^{2}+yz+z^{2})(z^{2}+zx+x^{2})\geq (x+y+z)^{2}(xy+yz+zx)^{2}$




#484433 Tìm GTNN của cosB.

Đã gửi bởi nam8298 on 23-02-2014 - 19:54 trong Bất đẳng thức - Cực trị

ta có cos B = $\frac{a^{2}+c^{2}-b^{2}}{2ac}$  (1)

do AA` cắt CC` tại trọng tâm tam giác .dùng công thức tính đường trung tuyến trong tam giác và dùng Py-ta-go thì điều kiện AA` vuông góc với CC` thì ta có $a^{2}+c^{2} = 5b^{2}$

thay vào (1) .áp dụng 2ac <= a^2 +c^2 thì tìm đc min cos B




#457046 $3(x^{2}+xy+y^{2})(y^{2}+yz+z^{2...

Đã gửi bởi nam8298 on 12-10-2013 - 12:38 trong Bất đẳng thức và cực trị

x,y,z  là các số thực chứ có phải dương đâu mà làm thế đc81(x+y)2(y+z)2(z+x)264(x+y+z)2(xy+yz+xz)29(x+y)(y+z)(z+x)8(x+y+z)(xy+yz+zx)




#468592 (3a+2b+c)(\frac{1}{a}+\frac{1}{b...

Đã gửi bởi nam8298 on 03-12-2013 - 18:57 trong Bất đẳng thức và cực trị

Ta đi cm 2 bbđt phụ sau:  Với a,b,c thuộc [1,2] thì  $\left ( a+b+c \right )\left ( \frac{1}{a}+\frac{1}{b}+\frac{1}{c} \right )\leq 10$  và $9\left ( a+b+c \right )\geq 4\left ( 3a+2b+c \right )$

 chứng minh ý 2 kiểu gì vậy bạn




#468400 (3a+2b+c)(\frac{1}{a}+\frac{1}{b...

Đã gửi bởi nam8298 on 02-12-2013 - 19:26 trong Bất đẳng thức và cực trị

Cho $a,b,c \in [1;2] .CMR (3a+2b+c)(\frac{1}{a}+\frac{1}{b}+\frac{1}{c})\leq \frac{45}{2}$




#478086 Cho a,b,c không âm thoả mãn

Đã gửi bởi nam8298 on 19-01-2014 - 19:48 trong Bất đẳng thức và cực trị

Bài 2; Đặt $\frac{1}{x}=a ;\frac{1}{y}=b;\frac{1}{z}=c$

BĐT càn chứng minh tương đương với $a+b+c +\sum \sqrt{(a+b)(a+c)}\leq \frac{1}{abc}\Leftrightarrow abc(a+b+c)+abc \sum \sqrt{(a+b)(a+c)}\leq 1$

Áp dụng AM-GM ta có $abc\sum \sqrt{(a+b)(a+c)}\leq 2(a+b+c)abc$

nên BĐT cần chứng minh tương đương 3abc(a+b+c)$\leq 1$ (luôn đúng do ab+bc+ca=1)




#489270 $\sum \frac{a}{b+c}\geqslant \fr...

Đã gửi bởi nam8298 on 28-03-2014 - 20:32 trong Bất đẳng thức - Cực trị

dùng SOS

bđt cần chứng minh tương đương với $\sum \frac{a}{b+c}-\frac{3}{2}\geq \frac{2}{3}(1-\frac{ab+bc+ca}{a^{2}+b^{2}+c^{2}})$

do  $\sum \frac{a}{b+c}-\frac{3}{2}= \sum \frac{(a-b)^{2}}{2(b+c)(c+a)}$

      $\frac{2}{3}(1-\frac{ab+bc+ca}{a^{2}+b^{2}+c^{2}})= \sum \frac{(a-b)^{2}}{3(a^{2}+b^{2}+c^{2})}$

vì  3(a^2 + b^2 + c^2 ) -2(c+a)(c+b) = (a+b-c)^2 +2(a-b)^2 >= 0 nên bđt đc chứng minh




#477088 Tứ Giác Đều Hòa

Đã gửi bởi nam8298 on 13-01-2014 - 18:54 trong Hình học

đây là tíng chất cơ bản của tứ giác điều hòa mà bạn




#461134 $\frac{a+b}{ab+a+b}+\frac{b+c}...

Đã gửi bởi nam8298 on 31-10-2013 - 20:02 trong Bất đẳng thức - Cực trị

Cho a,b,c >0 thỏa mãn $a^{2}+b^{2}+c^{2}= 1$  .CMR $\frac{a+b}{ab+1}+\frac{b+c}{bc+1}+\frac{c+a}{ca+1}\leq \frac{9}{2(a+b+c)}$




#471623 Tìm giá trị nhỏ nhất của A =\sqrt{(x+y)(y+z)(z+x)}.(\frac...

Đã gửi bởi nam8298 on 18-12-2013 - 20:26 trong Bất đẳng thức và cực trị

cái x ở mẫu cho nó vào trong




#471615 Tìm giá trị nhỏ nhất của A =\sqrt{(x+y)(y+z)(z+x)}.(\frac...

Đã gửi bởi nam8298 on 18-12-2013 - 20:13 trong Bất đẳng thức và cực trị

áp dụng Cauchy-Schwazt ta có A =$\sum (y+z)\sqrt{(1+\frac{y}{x})(1+\frac{z}{x})}\geq \sum (y+z)(1+\frac{\sqrt{yz}}{x})\geq \sum (y+z)+\sum \frac{2yz}{x}\geq 3(x+y+z)= 3\sqrt{2}$




#466328 Đề thi chọn đội tuyển trường THCS Bạch Liêu

Đã gửi bởi nam8298 on 23-11-2013 - 20:50 trong Tài liệu - Đề thi

1, Tìm tất cả các số nguyên dương a,b,c đôi một khác nhau sao cho:

$A=\frac{1}{a}+\frac{1}{b}+\frac{1}{c}+\frac{1}{ab}+\frac{1}{bc}+\frac{1}{ca}$ nhận giá trị nguyên dương.

2 Chứng minh rằng:

$\frac{1}{2}+\frac{1}{3\sqrt{2}}+...+\frac{1}{(n+1)\sqrt{n}}\leq 2$

3.Cho x,y,z>0 và $x+y+z\geq 1$.Chứng minh:

$\frac{x^3}{y^2}+\frac{y^3}{z^2}+\frac{z^3}{x^2}\geq 1$

4.Cho tam giác ABC. Trên cạnh BC lấy 2 điểm M,N. Chứng minh rằng:

$\widehat{MAB}=\widehat{NAC}\Leftrightarrow \frac{MB.NB}{MC.NC}=(\frac{AB}{AC})^2$.

bà 1 : giả sử $a\geq b\geq c$ .nếu c$c\geq 3$ thì A < 1 nên c=1 hoặc c=2 .

c=1 .làm tương tự chặn đc a và b

c=2 cũng tương tự




#471627 BĐT $\frac{6(a+b+c)(ab+bc+ca)}{(a+b)(b+c)(c+a)}...

Đã gửi bởi nam8298 on 18-12-2013 - 20:34 trong Bất đẳng thức - Cực trị

1.a,b,c >0 .CMR $\frac{6(a+b+c)(ab+bc+ca)}{(a+b)(b+c)(c+a)}+\frac{(a+b)^{2}(b+c)^{2}(c+a)^{2}}{abc(a+b+c)^{3}}\geq \frac{985}{108}$

2.x,y,z > 0 thỏa mãn xy+yz+zx =1.Tìm min  $\sum \frac{x^{2}}{1+x(x+\sqrt{1+x^{2}})}$